Tài liệu hạn chế xem trước, để xem đầy đủ mời bạn chọn Tải xuống
1
/ 51 trang
THÔNG TIN TÀI LIỆU
Thông tin cơ bản
Định dạng
Số trang
51
Dung lượng
301 KB
Nội dung
Problem 1 A convex quadrilateral ABCD has perpendicular diagonals. The perpendicular bisectors of AB and CD meet at a unique point P inside ABCD. Prove that ABCD is cyclic if and only if triangles ABP and CDP have equal areas. Solution: Let AC and BD intersect at E. Suppose by symmetry that P is in ABE. Denote by M and N the respective feet of perpendiculars from P to AC and BD. Without assuming that PA PB and PC PD, we express the areas ABP and CDP as follows: 2ABP 2ABE 2PAE 2PBE AM PNBN PM AM PNPM BN PMPN AM BN PM PN, 2CDP 2CDE 2PCE 2PDE CM PNDN PM CM PNPM DN PMPN CM DN PM PN. Therefore 2ABP CDP AM BN CM DN. We now assume that PA PB and PC PD. Suppose that ABCD is cyclic. Then the uniqueness of P implies that it must be the circumcenter. So M and N are the midpoints of AC and BD, respectively. Hence AM CM and BN DN,so(*) implies ABP CDP. Conversely, suppose that ABP CDP. Then, by (*), we have AM BN CM DN.IfPA PC, assume by symmetry that PA PC. Then AM CM and also BN DN, because PB PD. Thus AM BN CM DN, a contradiction. Hence PA PC, which implies that P is equidistant from A, B, C and D. We conclude then that ABCD is cyclic. Alternative Solution: Let AC and BD meet at E again. Assume by symmetry that P lies in BEC and denote ABE and ACD . The triangles ABP and CDP are isosceles. If M and N are the respective midpoints of their bases AB and CD, then PM AB and PN CD. Note that M, N and P are not collinear due to the uniqueness of P. Consider the median EM to the hypotenuse of the right triangle ABE.Wehave BEM , AME 2 and EMP 90 2. Likewise, CEN , DNE 2 and ENP 90 2. Hence MEN 90 , and a direct computation yields NPM 360 EMP MEN ENP 90 MEN. It turns out that, whenever AC BD, the quadrilateral EMPN has a pair of equal opposite angles, the ones at E and P. We now prove our claim. Since AB 2EM and CD 2EN,wehaveABP CDP if and only if EM PM EN PN,or,EM/EN PN/PM. On account of MEN NPM, the latter is equivalent to EMN PNM. This holds if and only if EMN PNM and ENM PMN, and these in turn mean that EMPN is a parallelogram. But the opposite angles of EMPN at E and P are always equal, as noted above. So it is a parallelogram if and only if EMP ENP;thatis,if 90 2 90 2. We thus obtain a condition equivalent to ,ortoABCD being cyclic. Another Solution: Choose a coordinate system so that the axes lie on the perpendicular lines AC and BD, and so that the coordinates of A, B, C and D are 0,a, b,0, 0,c and d,0, respectively. By assumption, the perpendicular bisectors of AB and CD have a unique common point. Hence the linear system formed by their equations 2bx 2ay b 2 a 2 and 2dx 2cy d 2 c 2 has a unique solution, and it is given by x 0 cb 2 a 2 ad 2 c 2 2ad bc , y 0 db 2 a 2 bd 2 c 2 2ad bc . Naturally, x 0 ,y 0 are the coordinates of P. Since it is interior to ABCD, ABP and CDP have the same orientation. Then ABP CDP if and only if || || This is equivalent to ax 0 by 0 ab cx 0 dy 0 cd. Inserting the expressions for x 0 and y 0 , after the inevitable algebra work we obtain the equivalent condition ac bda c 2 b d 2 0. Now, the choice of the coordinate system implies that a and c have different signs, as well as b and d. Hence the second factor is nonzero, so ABP CDP if and only if ac bd. The latter is equivalent to AE CE BE DE, where E is the intersection point of the diagonals. However, it is a necessary and sufficient condition for ABCD to be cyclic. Problem 2 Let ABCD be a cyclic quadrilateral. Let E and F be variable points on the sides AB and CD, respectively, such that AE : EB CF : FD.LetP be the point on the segment EF such that PE : PF AB : CD. Prove that the ratio between the areas of triangles APD and BPC does not depend on the choice of E and F. Solution: We first assume that the lines AD and BC are not parallel and meet at S.SinceABCD is cyclic, ASB and CSD are similar. Then, since AE : EB CF : FD, ASE and CSF are also similar, so that DSE CSF. Moreover, we have SE SF SA SC AB CD PE PF , which implies that ESP FSP. Thus, ASP BSP,andsoP is equidistant from the lines AD and BC. Therefore, APD : BPC AD : BC, which is a constant. Next, assume that the lines AD and BC are parallel. Then ABCD is an isosceles trapezoid with AB CD, and we have BE DF.LetM and N be the midpoints of AB and CD, respectively. Then ME NF and clearly, E and F are equidistant from the line MN. Thus P, the midpoint of EF, lies on MN. It follows that P is equidistant from AD and BC, and hence APD : BPC AD : BC. Alternative Solution: Let AE : EB CF : FD a : b,wherea b 1. Since PE : PF AB : CD,we have d P,AD CD AB CD d E,AD AB AB CD d F,AD , d P,BC CD AB CD d E,BC AB AB CD d F,BC , where dX,YZ stands for the distance from the point X to the line YZ. Thus, we obtain APD CD AB CD AED AB AB CD AFD a CD AB CD ABD b AB AB CD ACD, BPC CD AB CD BEC AB AB CD BFC b CD AB CD BAC a AB AB CD BDC. Next, since A, B, C and D areconcyclic,wehavesinBAD sinBCD and sinABC sinADC. Thus, APD BPC a CD ABD b AB ACD b CD BAC a AB BDC a CD AB AD sinBAD b AB CD AD sinADC b CD AB BC sinABC a AB CD BC sinBCD AD BC a sinBAD b sinADC b sinABC a sinBCD AD BC . Problem 3 Let I be the incenter of triangle ABC.LetK,L and M be the points of tangency of the incircle of ABC with AB,BC and CA, respectively. The line t passes through B and is parallel to KL. The lines MK and ML intersect t at the points R and S. Prove that RIS is acute. Solution: Since the lines KL and RS are parallel, we have, in BKR , BKR 90° A/2, KBR 90° B/2, BRK 90° C/2. Hence, by the law of sine, BR cosA/2 cosC/2 BK. # Similarly, we have, in BLS, BLS 90° C/2, BSL 90° A/2, LBS 90° B/2. so that BS cos C/2 cos A/2 BL cos C/2 cos A/2 BK. # Notice now that BI RS and IK AB. Then, on account of (1) and (2), we obtain IR 2 IS 2 RS 2 BI 2 BR 2 BI 2 BS 2 BR BS 2 2 BI 2 BR BS 2 BI 2 BK 2 2IK 2 0. So,bythelawofcosine,RIS is acute. Alternative Solution: Let W be the midpoint of KL and Q the midpoint of KM. Then Q AI, W BI, AIKM and BIKL. We first prove that AWRI and CWSI. Since RBI RQI 90 , the points R, B, I, Q are concyclic, which implies BRI BQI. Also, in the right triangles AIK and BIK,wehave IQ IA KI 2 IW IB, so that IQ/IW IB/IA. Hence AIW and BIQ are similar. It follows that BRI BQI AWI.SinceBR IW, this implies that RIAW.Byasimilar argument, we can prove that SICW. Thus, RIS 180 AWC. It remains to prove that AWC is obtuse. Let T be the midpoint of AC.Then2 WT WC WA LC KA.SinceLC and KA are not collinear, we have WT LC KA 2 CM AM 2 AC 2 . This implies that W is inside the circle with diameter AC,andsoAWC 90°. Therefore, RIS is acute. Comment: Another proof for the fact AW RI is as follows. Since RBI RQI 90°, RBIQ is cyclic and its circumcircle is orthogonal to the diameter RI. Consider the inversion with respect to the incircle of ABC.Since takes B and Q into W and A, respectively, it takes the circumcircle of RBIQ into the line AW.Since leaves the line RI invariant, we have AW RI. Problem 4 Let M and N be points inside triangle ABC such that MAB NAC MBA NBC. Prove that AM AN AB AC BM BN BA BC CM CN CA CB 1. Solution: Let K be the point on the ray BN such that BCK BMA. Note that K is outside ABC, because BMA ACB.SinceMBA CBK,wehaveABM KBC;so, AB BK BM BC AM CK . # Then, since ABK MBC,andAB/KB BM/BC, we see that ABK MBC. Hence AB BM BK BC AK CM . # Now we have CKN MAB NAC. Consequently, the points A, N, C and K are concyclic. By Ptolemy’s theorem, AC NK AN CK CN AK,or AC BK BN AN CK CN AK. # From (1) and (2), we find CK AM BC/BM, AK AB CM/BM and BK AB BC/BM. Inserting these expressions in (3), we obtain AC AB BC BM BN AN AM BC BM CN AB CM BM , or AM AN AB AC BM BN BA BC CM CN CA CB 1. Alternative Solution: Let the complex coordinates of A, B, C, M and N be a, b, c, m and n, respectively. Since the lines AM, BM and CM are concurrent, as well as the lines AN, BN and CN, it follows from Ceva’s theorem that sinBAM sinMAC sinCBM sinMBA sinACM sinMCB 1. # sinBAN sinNAC sinCBN sinNBA sinACN sinNCB 1. # By hypotheses, BAM NAC and MBA CBN. Hence BAN MAC and NBA CBM. Combined with (1) and (2), these equalities imply sinACM sinACN sinMCB sinNCB. Thus, cos NCM 2ACM cosNCM cos NCM 2NCB cosNCM. and hence ACM NCB. Since BAM NAC, MBA CBN and ACN MCB, the following complex ratios are all positive real numbers: m a b a : c a n a , m b a b : c b n b and m c b c : a c n c . Hence each of these equals its absolute value, and so AM AN AB AC BM BN BA BC CM CN CA CB m a n a b a c a m b n b a b c b m c n c b c a c 1. Comment: Contestants who are familiar with the notion of isogonal conjugate points may skip over the early part of the proof. Problem 5 Let ABC be a triangle, H its orthocenter, O its circumcenter, and R its circumradius. Let D be the reflection of A across BC, E be that of B across CA,andF that of C across AB. Prove that D,E and F are collinear if and only if OH 2R. Solution: Let G be the centroid of ABC,andA , B and C be the midpoints of BC, CA and AB, respectively. Let A B C be the triangle for which A, B and C are the midpoints of B C , C A and A B , respectively. Then G is the centroid and H the circumcenter of A B C . Let D , E and F denote the projections of O on the lines B C , C A and A B , respectively. Consider the homothety h with center G and ratio 1/2. It maps A, B, C, A , B and C into A , B , C , A , B and C, respectively. Note that A D BC, which implies AD : A D 2:1 GA : GA and DAG D A G. We conclude that hD D . Similarly, h E E and h F F . Thus, D, E and F are collinear if and only if D , E and F are collinear. Now D , E and F are the projections of O on the sides B C , C A and A B , respectively. By Simson’s theorem, they are collinear if and only if O lies on the circumcircle of A B C . Since the circumradius of A B C is 2R, O lies on its circumcircle if and only if OH 2R. Alternative Solution: Let the complex coordinates of A, B, C, H and O be a, b, c, h and 0, respectively. Consequently, aa bb cc R 2 and h a b c.SinceD is symmetric to A with respect to line BC, the complex coordinates d and a satisfy d b c b a b c b ,or b c d b c a bc b c 0. # Since b c R 2 b c bc and bc b c R 2 b 2 c 2 bc , by inserting these expressions in (1), we obtain d bc ca ab a k 2bc a . d R 2 a b c bc R 2 h 2a bc . where k bc ca ab. Similarly, we have e k 2ca b , e R 2 h 2b ca , f k 2ab c andf R 2 h 2c ab . Since dd 1 ee 1 ff 1 e de d f df d ba k2ab ab R 2 ab h2c abc ca k2ca ca R 2 ac h2b abc R 2 c a a b a 2 b 2 c 2 ck 2abc h 2c bk 2abc h 2b R 2 b c c a a b hk 4abc a 2 b 2 c 2 and h R 2 k/abc, it follows that D,EandFarecollinear 0 hk 4abc 0 hh 4R 2 OH 2R. [...]... ggn n), and so takes distinct primes to distinct primes Hence a lower bound for g 1998 g2 3 3 37 g2g3 3 g37 is obtained when g2, g3, g37 are the three smallest primes 2, 3, 5, with g3 2 Thus, g 1998 3 2 3 5 120 for each g S There is, however, a function g S with g 1998 120; this proves that the minimum in question is 120 Set g1 1, and define g... 10 and 6m 10 1mod3 is already in the list Finally, we have a 4m8 9m 16, becsase no y 2mod3 is in the list The formulae for a n has been established by induction From 1998 4 499 2, we find that a 1998 9 499 3 4494 Comment: It is easy to see that x y 3z has no solution in the set of positive integers 1 Multiply the set by 3 still has the same property Thus one takes... arbitrary n p 1 p 2 p k N by setting 1 2 k gn gp 1 p 2 p k gp 1 1 gp 2 2 gp k k The conditions in (2) are satisfied 1 2 k (with a 1), so g S Clearly, g 1998 120, which completes the proof Comment: 1 Actually, each function f considered above is of the form fn agn, where g is a multiplicative involution of N; that is, a function satisfying (2)... is defined as follows: a 1 1 and for n 1, a n1 is the smallest integer greater than a n such that a i a j 3a k for any i, j and k in 1, 2, 3, , n 1, not necessarily distinct Determine a 1998 Solution: We investigate the first few values for a n , writing ‘not m’ to mean that the next value for a n cannot be m We have a 1 1, but not 2, because 1 2 3 1 We next find that a 2 3,... generating polynomials f n x does not work in itself To prove the mentioned algebraic relation presents no less difficulty than the original problem Problem 13 Determine the least possible value of f 1998 , where f is a function from the set N of positive integers into itself such that for all m, n N, fn 2 fm mfn 2 Solution: Denote by S the set of functions considered Let f be any of them,... consisting of positive integers 1, 3, 4, 7 Ordering this set as an increasing sequence a n The subtle point is to show that this sequence indeed is what we want, then it is easy to get the value of a 1998 from the formula for a n Problem 18 Determine all positive integers n for which there exists an integer m such that 2 n 1 is a divisor of m 2 9 Solution: First we show that if 2 n 1 divides . 1. Comment: Contestants who are familiar with the notion of isogonal conjugate points may skip over the early part of the proof. Problem 5 Let ABC be a triangle, H its orthocenter, O its circumcenter,. expressions for x 0 and y 0 , after the inevitable algebra work we obtain the equivalent condition ac bda c 2 b d 2 0. Now, the choice of the coordinate system implies that a and c. being cyclic. Another Solution: Choose a coordinate system so that the axes lie on the perpendicular lines AC and BD, and so that the coordinates of A, B, C and D are 0,a, b,0, 0,c and